Difference between revisions of "2010 AIME I Problems/Problem 15"
(→Solution 2) |
|||
(26 intermediate revisions by 13 users not shown) | |||
Line 1: | Line 1: | ||
__TOC__ | __TOC__ | ||
== Problem == | == Problem == | ||
− | In <math>\triangle{ABC}</math> with <math>AB = 12</math>, <math>BC = 13</math>, and <math>AC = 15</math>, let <math>M</math> be a point on <math>\overline{AC}</math> such that the [[incircle]]s of <math>\triangle{ABM}</math> and <math>\triangle{BCM}</math> have equal [[inradius|radii]]. | + | In <math>\triangle{ABC}</math> with <math>AB = 12</math>, <math>BC = 13</math>, and <math>AC = 15</math>, let <math>M</math> be a point on <math>\overline{AC}</math> such that the [[incircle]]s of <math>\triangle{ABM}</math> and <math>\triangle{BCM}</math> have equal [[inradius|radii]]. Then <math>\frac{AM}{CM} = \frac{p}{q}</math>, where <math>p</math> and <math>q</math> are relatively prime positive integers. Find <math>p + q</math>. |
− | |||
== Solution == | == Solution == | ||
Line 31: | Line 30: | ||
label("$12$",(2.28834,5.75684),NE); | label("$12$",(2.28834,5.75684),NE); | ||
dot((7.33333,0)); | dot((7.33333,0)); | ||
− | label("$M$",(7.56053,- | + | label("$M$",(7.56053,-1.000),NE); |
+ | label("$H_1$",(3.97942,-1.200),NE); | ||
+ | label("$H_2$",(9.54741,-1.200),NE); | ||
dot((4.66667,2.49444)); | dot((4.66667,2.49444)); | ||
label("$I_1$",(3.97942,2.92179),NE); | label("$I_1$",(3.97942,2.92179),NE); | ||
dot((9.66667,2.49444)); | dot((9.66667,2.49444)); | ||
− | label("$I_2$",( | + | label("$I_2$",(9.54741,2.92179),NE); |
clip((-3.72991,-6.47862)--(-3.72991,17.44518)--(32.23039,17.44518)--(32.23039,-6.47862)--cycle); | clip((-3.72991,-6.47862)--(-3.72991,17.44518)--(32.23039,17.44518)--(32.23039,-6.47862)--cycle); | ||
</asy></center> | </asy></center> | ||
=== Solution 1 === | === Solution 1 === | ||
− | Let <math>AM = x</math>, then <math>CM = 15 - x</math>. Also let <math>BM = d</math> Clearly, <math>\frac {[ABM]}{[CBM]} = \frac {x}{15 - x}</math>. We can also express each area by the rs formula. Then <math>\frac {[ABM]}{[CBM]} = \frac {p(ABM)}{p(CBM)} = \frac {12 + d + x}{28 + d - x}</math>. Equating and cross-multiplying yields <math>25x + 2dx = 15d + 180</math> or <math>d = \frac {25x - 180}{15 - 2x}.</math> Note that for d to be positive, we must have <math>7.2 < x < 7.5</math>. | + | Let <math>AM = x</math>, then <math>CM = 15 - x</math>. Also let <math>BM = d</math> Clearly, <math>\frac {[ABM]}{[CBM]} = \frac {x}{15 - x}</math>. We can also express each area by the rs formula. Then <math>\frac {[ABM]}{[CBM]} = \frac {p(ABM)}{p(CBM)} = \frac {12 + d + x}{28 + d - x}</math>. Equating and cross-multiplying yields <math>25x + 2dx = 15d + 180</math> or <math>d = \frac {25x - 180}{15 - 2x}.</math> Note that for <math>d</math> to be positive, we must have <math>7.2 < x < 7.5</math>. |
By [[Stewart's Theorem]], we have <math>12^2(15 - x) + 13^2x = d^215 + 15x(15 - x)</math> or <math>432 = 3d^2 + 40x - 3x^2.</math> Brute forcing by plugging in our previous result for <math>d</math>, we have <math>432 = \frac {3(25x - 180)^2}{(15 - 2x)^2} + 40x - 3x^2.</math> Clearing the fraction and gathering like terms, we get <math>0 = 12x^4 - 340x^3 + 2928x^2 - 7920x.</math> | By [[Stewart's Theorem]], we have <math>12^2(15 - x) + 13^2x = d^215 + 15x(15 - x)</math> or <math>432 = 3d^2 + 40x - 3x^2.</math> Brute forcing by plugging in our previous result for <math>d</math>, we have <math>432 = \frac {3(25x - 180)^2}{(15 - 2x)^2} + 40x - 3x^2.</math> Clearing the fraction and gathering like terms, we get <math>0 = 12x^4 - 340x^3 + 2928x^2 - 7920x.</math> | ||
− | ''Aside: Since <math>x</math> must be rational in order for our answer to be in the desired form, we can use the [[Rational Root Theorem]] to reveal that <math> | + | ''Aside: Since <math>x</math> must be rational in order for our answer to be in the desired form, we can use the [[Rational Root Theorem]] to reveal that <math>6x</math> is an integer because we can divide the polynomial by <math>2</math>. The only such <math>x</math> in the above-stated range is <math>\frac {22}3</math>.'' |
Legitimately solving that quartic, note that <math>x = 0</math> and <math>x = 15</math> should clearly be solutions, corresponding to the sides of the triangle and thus degenerate cevians. Factoring those out, we get <math>0 = 4x(x - 15)(3x^2 - 40x + 132) = x(x - 15)(x - 6)(3x - 22).</math> The only solution in the desired range is thus <math>\frac {22}3</math>. Then <math>CM = \frac {23}3</math>, and our desired ratio <math>\frac {AM}{CM} = \frac {22}{23}</math>, giving us an answer of <math>\boxed{045}</math>. | Legitimately solving that quartic, note that <math>x = 0</math> and <math>x = 15</math> should clearly be solutions, corresponding to the sides of the triangle and thus degenerate cevians. Factoring those out, we get <math>0 = 4x(x - 15)(3x^2 - 40x + 132) = x(x - 15)(x - 6)(3x - 22).</math> The only solution in the desired range is thus <math>\frac {22}3</math>. Then <math>CM = \frac {23}3</math>, and our desired ratio <math>\frac {AM}{CM} = \frac {22}{23}</math>, giving us an answer of <math>\boxed{045}</math>. | ||
=== Solution 2 === | === Solution 2 === | ||
− | Let <math>AM = 2x</math> and <math>BM = 2y</math> so <math>CM = 15 - 2x</math>. Let the [[incenter]]s of <math>\triangle ABM</math> and <math>\triangle BCM</math> be <math>I_1</math> and <math>I_2</math> respectively, and their equal inradii be <math>r</math>. From <math>r = \sqrt {(s - a)(s - b)(s - c)/s | + | Let <math>AM = 2x</math> and <math>BM = 2y</math> so <math>CM = 15 - 2x</math>. Let the [[incenter]]s of <math>\triangle ABM</math> and <math>\triangle BCM</math> be <math>I_1</math> and <math>I_2</math> respectively, and their equal inradii be <math>r</math>. From <math>r = \sqrt {(s - a)(s - b)(s - c)}/s</math>, we find that |
<cmath>\begin{align*}r^2 & = \frac {(x + y - 6)( - x + y + 6)(x - y + 6)}{x + y + 6} & (1) \\ | <cmath>\begin{align*}r^2 & = \frac {(x + y - 6)( - x + y + 6)(x - y + 6)}{x + y + 6} & (1) \\ | ||
Line 55: | Line 56: | ||
Let the incircle of <math>\triangle ABM</math> meet <math>AM</math> at <math>P</math> and the incircle of <math>\triangle BCM</math> meet <math>CM</math> at <math>Q</math>. Then note that <math>I_1 P Q I_2</math> is a rectangle. Also, <math>\angle I_1 M I_2</math> is right because <math>MI_1</math> and <math>MI_2</math> are the angle bisectors of <math>\angle AMB</math> and <math>\angle CMB</math> respectively and <math>\angle AMB + \angle CMB = 180^\circ</math>. By properties of [[tangent (geometry)|tangents]] to [[circle]]s <math>MP = (MA + MB - AB)/2 = x + y - 6</math> and <math>MQ = (MB + MC - BC)/2 = - x + y + 1</math>. Now notice that the altitude of <math>M</math> to <math>I_1 I_2</math> is of length <math>r</math>, so by similar triangles we find that <math>r^2 = MP \cdot MQ = (x + y - 6)( - x + y + 1)</math> (3). Equating (3) with (1) and (2) separately yields | Let the incircle of <math>\triangle ABM</math> meet <math>AM</math> at <math>P</math> and the incircle of <math>\triangle BCM</math> meet <math>CM</math> at <math>Q</math>. Then note that <math>I_1 P Q I_2</math> is a rectangle. Also, <math>\angle I_1 M I_2</math> is right because <math>MI_1</math> and <math>MI_2</math> are the angle bisectors of <math>\angle AMB</math> and <math>\angle CMB</math> respectively and <math>\angle AMB + \angle CMB = 180^\circ</math>. By properties of [[tangent (geometry)|tangents]] to [[circle]]s <math>MP = (MA + MB - AB)/2 = x + y - 6</math> and <math>MQ = (MB + MC - BC)/2 = - x + y + 1</math>. Now notice that the altitude of <math>M</math> to <math>I_1 I_2</math> is of length <math>r</math>, so by similar triangles we find that <math>r^2 = MP \cdot MQ = (x + y - 6)( - x + y + 1)</math> (3). Equating (3) with (1) and (2) separately yields | ||
− | <cmath> | + | <cmath>\begin{align*} |
2y^2 - 30 = 2xy + 5x - 7y \\ | 2y^2 - 30 = 2xy + 5x - 7y \\ | ||
− | 2y^2 - 70 = - 2xy - 5x + 7y, | + | 2y^2 - 70 = - 2xy - 5x + 7y, \end{align*} |
</cmath> | </cmath> | ||
Line 75: | Line 76: | ||
Now, we get <math>BM=BZ_{2}+Z_{2}M=BZ_{2}+MX_{2}=20-15x</math>. To finish, we will compute area ratios. <math>\dfrac{[ABM]}{[CBM]}=\dfrac{AM}{MC}=\dfrac{8-x}{7+x}</math>. Also, since their inradii are equal, we get <math>\dfrac{[ABM]}{[CBM]}=\dfrac{40-16x}{40-14x}</math>. Equating and cross multiplying yields the quadratic <math>3x^{2}-8x+4=0</math>, so <math>x=2/3,2</math>. However, observe that <math>AX_{1}+CX_{2}=15x<15</math>, so we take <math>x=2/3</math>. Our ratio is therefore <math>\dfrac{8-2/3}{7+2/3}=\dfrac{22}{23}</math>, giving the answer <math>\boxed{045}</math>. | Now, we get <math>BM=BZ_{2}+Z_{2}M=BZ_{2}+MX_{2}=20-15x</math>. To finish, we will compute area ratios. <math>\dfrac{[ABM]}{[CBM]}=\dfrac{AM}{MC}=\dfrac{8-x}{7+x}</math>. Also, since their inradii are equal, we get <math>\dfrac{[ABM]}{[CBM]}=\dfrac{40-16x}{40-14x}</math>. Equating and cross multiplying yields the quadratic <math>3x^{2}-8x+4=0</math>, so <math>x=2/3,2</math>. However, observe that <math>AX_{1}+CX_{2}=15x<15</math>, so we take <math>x=2/3</math>. Our ratio is therefore <math>\dfrac{8-2/3}{7+2/3}=\dfrac{22}{23}</math>, giving the answer <math>\boxed{045}</math>. | ||
+ | |||
+ | Note: Once we have <math>MX_1=8-8x</math> and <math>MX_2=7-7x</math>, it's bit easier to use use the right triangle of <math>O_1MO_2</math> than chasing the area ratio. The inradius of <math>\triangle{ABC}</math> can be calculated to be <math>r=\sqrt{14}</math>, and the inradius of <math>ABM</math> and <math>ACM</math> are <math>r_1=r_2= xr</math>, so, | ||
+ | <cmath> O_1O_2^2 = O_1M^2 + O_2M^2 = r_1^2+X_1M^2 + r_2^2 + X_2M^2</cmath> | ||
+ | or, | ||
+ | <cmath> (15(1-x))^2 = 2(\sqrt{14}x)^2 + (7(1-x))^2 + (8(1-x))^2 </cmath> | ||
+ | <cmath>112(1-x)^2 = 28x^2</cmath> | ||
+ | <cmath>4(1-x)^2 = x^2</cmath> | ||
+ | We get <math>x=\frac{2}{3}</math> or <math>x=2</math>. | ||
=== Solution 4 === | === Solution 4 === | ||
Line 105: | Line 114: | ||
So <math>AM=22/3</math> or <math>6</math>. But from (1) we know that <math>5+15(AM-7)>0</math>, or <math>AM>7-1/3>6</math>, so <math>AM=22/3</math>, <math>CM=15-22/3=23/3</math>, <math>AM/CM=22/23</math>. | So <math>AM=22/3</math> or <math>6</math>. But from (1) we know that <math>5+15(AM-7)>0</math>, or <math>AM>7-1/3>6</math>, so <math>AM=22/3</math>, <math>CM=15-22/3=23/3</math>, <math>AM/CM=22/23</math>. | ||
+ | |||
+ | === Solution 5 === | ||
+ | |||
+ | Let the common inradius equal r, <math>BM = x</math>, <math>AM = y</math>, <math>MC = z</math> | ||
+ | |||
+ | From the prespective of <math>\triangle{ABM}</math> and <math>\triangle{BMC}</math> we get: | ||
+ | |||
+ | <math>S_{ABM} = rs = r \cdot (\frac{12+x+y}{2})</math> <math>S_{BMC} = rs = r \cdot (\frac{13+x+z}{2})</math> | ||
+ | |||
+ | Add two triangles up, we get <math>\triangle{ABC}</math> : | ||
+ | |||
+ | <math>S_{ABC} = S_{ABM} + S_{BMC} = r \cdot \frac{25+2x+y+z}{2}</math> | ||
+ | |||
+ | Since <math>y + z = 15</math>, we get: | ||
+ | |||
+ | <math>r = \frac{S_{ABC}}{20 + x}</math> | ||
+ | |||
+ | By drawing an altitude from <math>I_1</math> down to a point <math>H_1</math> and from <math>I_2</math> to <math>H_2</math>, we can get: | ||
+ | |||
+ | <math>r \cdot cot(\frac{\angle A}{2}) =r \cdot A H_1 = r \cdot \frac{12+y-x}{2} </math> and | ||
+ | |||
+ | <math>r \cdot cot(\frac{\angle C}{2}) = r \cdot H_2 C = r \cdot \frac{13+z-x}{2}</math> | ||
+ | |||
+ | Adding these up, we get: | ||
+ | |||
+ | <math>r \cdot (cot(\frac{\angle A}{2})+cot(\frac{\angle C}{2})) = \frac{25+y+z-2x}{2} = \frac{40-2x}{2} = 20-x</math> | ||
+ | |||
+ | <math>r = \frac{20-x}{cot(\frac{\angle A}{2})+cot(\frac{\angle C}{2})}</math> | ||
+ | |||
+ | Now, we have 2 values equal to r, we can set them equal to each other: | ||
+ | |||
+ | <math>\frac{S_{ABC}}{20 + x} = \frac{20-x}{cot(\frac{\angle A}{2})+cot(\frac{\angle C}{2})}</math> | ||
+ | |||
+ | If we let R denote the incircle of ABC, note: | ||
+ | |||
+ | AC = <math>(cot(\frac{\angle A}{2})+cot(\frac{\angle C}{2})) \cdot R = 15</math> and | ||
+ | |||
+ | <math>S_{ABC} = \frac{12+13+15}{2} \cdot R = 20 \cdot R</math>. | ||
+ | |||
+ | By cross multiplying the equation above, we get: | ||
+ | |||
+ | <math>400 - x^2 = (cot(\frac{\angle A}{2})+cot(\frac{\angle C}{2})) \cdot S_{ABC} = (cot(\frac{\angle A}{2})+cot(\frac{\angle C}{2})) \cdot R \cdot 20 = 15 \cdot 20 = 300</math> | ||
+ | |||
+ | We can find out x: | ||
+ | |||
+ | <math>x = 10</math>. | ||
+ | |||
+ | Now, we can find ratio of y and z: | ||
+ | |||
+ | <math>\frac{AM}{CM} = \frac{y}{z} = \frac{S_{ABM}}{S_{BCM}} = \frac{r \cdot \frac{22+y}{2} }{r \cdot \frac{23+z}{2}} = \frac{22+y}{23+z} = \frac{22}{23}</math> | ||
+ | |||
+ | The answer is <math>\boxed{045}</math>. | ||
+ | |||
+ | -Alexlikemath | ||
+ | |||
+ | === Solution 6 (Similar to Solution 1 with easier computation) === | ||
+ | |||
+ | Let <math>CM=x, AM=rx, BM=d</math>. <math>x+rx=15\Rightarrow x=\frac{15}{1+r}</math>. | ||
+ | |||
+ | Similar to Solution 1, we have | ||
+ | <cmath> | ||
+ | r=\frac{[AMB]}{[CMB]}=\frac{12+rx+d}{13+x+d} \Rightarrow d=\frac{13r-12}{1-r} | ||
+ | </cmath> | ||
+ | as well as | ||
+ | <cmath> | ||
+ | 12^2\cdot x + 13^2 rx=15x\cdot rx+15d^2 (\text{via Stewart's Theorem}) | ||
+ | </cmath> | ||
+ | <cmath> | ||
+ | \frac{(12^2 + 13^2r) \cdot 15}{1+r} - \frac{15r\cdot 15^2}{(1+r)^2}=\frac{15(13r-12)^2}{(1-r)^2} | ||
+ | </cmath> | ||
+ | <cmath> | ||
+ | \frac{169r^2+88r+144}{(1+r)^2}=\frac{(13r-12)^2}{(1-r)^2} | ||
+ | </cmath> | ||
+ | <cmath> | ||
+ | (169r^2+88r+144)((r^2+1)-2r)=(169r^2-312r+144)((r^2+1)+2r) | ||
+ | </cmath> | ||
+ | <cmath> | ||
+ | (r^2+1)(400r)=2r(338r^2-224r+288) | ||
+ | </cmath> | ||
+ | <cmath> | ||
+ | 100(r^2+1)=169r^2-112r+144 \Rightarrow 69r^2-112r+44=(23r-22)(3r-2)=0 | ||
+ | </cmath> | ||
+ | |||
+ | Since <math>d=\frac{13r-12}{1-r}>0</math>, we have <math>r=\frac{22}{23} \longrightarrow \boxed{045}</math>. | ||
+ | |||
+ | ~ asops | ||
+ | |||
+ | === Solution 7 (No Stewart's or trig, fast + clever) === | ||
+ | |||
+ | Let <math>BM = d, AM = x, CM = 15 - x</math>. Observe that we have the equation by the incircle formula: | ||
+ | <cmath>\frac{[ABM]}{12 + AM + MB} = \frac{[CBM]}{13 + CM + MB} \implies \frac{AM}{CM} = \frac{12 + MB}{13 + MB} \implies \frac{x}{15 - x} = \frac{12 + d}{13 + d}.</cmath> | ||
+ | Now let <math>X</math> be the point of tangency between the incircle of <math>\triangle ABC</math> and <math>AC</math>. Additionally, let <math>P</math> and <math>Q</math> be the points of tangency between the incircles of <math>\triangle ABM</math> and <math>\triangle CBM</math> with <math>AC</math> respectively. Some easy calculation yields <math>AX = 7, CX = 8</math>. By homothety we have | ||
+ | <cmath>\frac{AP}{7} = \frac{CQ}{8} \implies 8(AP) = 7(CQ) \implies 8(12 + x - d) = 7(13 + 15 - x - d) \implies d = 15x - 100.</cmath> | ||
+ | Substituting into the first equation derived earlier it is left to solve | ||
+ | <cmath>\frac{x}{15 - x} = \frac{15x - 88}{15x - 87} \implies 3x^2 - 40x + 132 \implies (x - 6)(3x - 22) = 0.</cmath> | ||
+ | Now <math>x = 6</math> yields <math>d = -10</math> which is invalid, hence <math>x = \frac{22}{3}</math> so <math>\frac{AM}{CM} = \frac{\frac{22}{3}}{15 - \frac{22}{3}} = \frac{22}{23}.</math> The requested sum is <math>22 + 23 = \boxed{45}</math>. ~blueprimes | ||
+ | |||
+ | ==Video Solution== | ||
+ | https://www.youtube.com/watch?v=UQVI0Q2PWZw&feature=youtu.be&fbclid=IwAR338IdppnZVuze4rzT0gm8G2NB_uIsmj175WgD6sa43gg3EhFAGm5bAvV0 | ||
+ | |||
+ | == Sidenote == | ||
+ | In the problem, <math>BM=10</math> and the equal inradius of the two triangles happens to be <math> \frac {2\sqrt{14}}{3}</math>. | ||
== See Also == | == See Also == | ||
− | + | ||
{{AIME box|year=2010|num-b=14|after=Last Problem|n=I}} | {{AIME box|year=2010|num-b=14|after=Last Problem|n=I}} |
Latest revision as of 20:19, 29 September 2024
Contents
Problem
In with , , and , let be a point on such that the incircles of and have equal radii. Then , where and are relatively prime positive integers. Find .
Solution
Solution 1
Let , then . Also let Clearly, . We can also express each area by the rs formula. Then . Equating and cross-multiplying yields or Note that for to be positive, we must have .
By Stewart's Theorem, we have or Brute forcing by plugging in our previous result for , we have Clearing the fraction and gathering like terms, we get
Aside: Since must be rational in order for our answer to be in the desired form, we can use the Rational Root Theorem to reveal that is an integer because we can divide the polynomial by . The only such in the above-stated range is .
Legitimately solving that quartic, note that and should clearly be solutions, corresponding to the sides of the triangle and thus degenerate cevians. Factoring those out, we get The only solution in the desired range is thus . Then , and our desired ratio , giving us an answer of .
Solution 2
Let and so . Let the incenters of and be and respectively, and their equal inradii be . From , we find that
Let the incircle of meet at and the incircle of meet at . Then note that is a rectangle. Also, is right because and are the angle bisectors of and respectively and . By properties of tangents to circles and . Now notice that the altitude of to is of length , so by similar triangles we find that (3). Equating (3) with (1) and (2) separately yields
and adding these we have
Solution 3
Let the incircle of hit , , at , and let the incircle of hit , , at . Draw the incircle of , and let it be tangent to at . Observe that we have a homothety centered at A sending the incircle of to that of , and one centered at taking the incircle of to that of . These have the same power, since they take incircles of the same size to the same circle. Also, the power of the homothety is .
By standard computations, and . Now, let and . We will now go around and chase lengths. Observe that . Then, . We also have , so and .
Observe now that . Also,. Solving, we get and (as a side note, note that , a result that I actually believe appears in Mandelbrot 1995-2003, or some book in that time-range).
Now, we get . To finish, we will compute area ratios. . Also, since their inradii are equal, we get . Equating and cross multiplying yields the quadratic , so . However, observe that , so we take . Our ratio is therefore , giving the answer .
Note: Once we have and , it's bit easier to use use the right triangle of than chasing the area ratio. The inradius of can be calculated to be , and the inradius of and are , so, or, We get or .
Solution 4
Suppose the incircle of touches at , and the incircle of touches at . Then
We have ,
, ,
Therefore
And since , ,
Now,
So or . But from (1) we know that , or , so , , .
Solution 5
Let the common inradius equal r, , ,
From the prespective of and we get:
Add two triangles up, we get :
Since , we get:
By drawing an altitude from down to a point and from to , we can get:
and
Adding these up, we get:
Now, we have 2 values equal to r, we can set them equal to each other:
If we let R denote the incircle of ABC, note:
AC = and
.
By cross multiplying the equation above, we get:
We can find out x:
.
Now, we can find ratio of y and z:
The answer is .
-Alexlikemath
Solution 6 (Similar to Solution 1 with easier computation)
Let . .
Similar to Solution 1, we have as well as
Since , we have .
~ asops
Solution 7 (No Stewart's or trig, fast + clever)
Let . Observe that we have the equation by the incircle formula: Now let be the point of tangency between the incircle of and . Additionally, let and be the points of tangency between the incircles of and with respectively. Some easy calculation yields . By homothety we have Substituting into the first equation derived earlier it is left to solve Now yields which is invalid, hence so The requested sum is . ~blueprimes
Video Solution
Sidenote
In the problem, and the equal inradius of the two triangles happens to be .
See Also
2010 AIME I (Problems • Answer Key • Resources) | ||
Preceded by Problem 14 |
Followed by Last Problem | |
1 • 2 • 3 • 4 • 5 • 6 • 7 • 8 • 9 • 10 • 11 • 12 • 13 • 14 • 15 | ||
All AIME Problems and Solutions |
The problems on this page are copyrighted by the Mathematical Association of America's American Mathematics Competitions.